Lösungen zu den Aufgaben aus Kapitel 7:

Differenzierbare Funktionen - Sätze über differenzierbare Funktionen


 

 

Lösungen zu den Aufgaben Der Satz von Rolle

 

Lösung zur Aufgabe 7.3.1 - Satz von Rolle und zusammengesetzte Funktionen

Der Satz von Rolle ist hier nicht anwendbar, d.h. es existiert kein \( \xi\in(0,1) \) mit \( f'(\xi)=0. \) Denn es ist \( f(x)=x, \) \( x\in(0,1), \) mit der Ableitung \( f'(x)=1 \) für alle \( x\in(0,1). \) Die Funktion ist in \( (0,1) \) zwar differenzierbar, aber sie ist auf \( [0,1] \) nicht stetig. Die Voraussetzungen des Satzes von Rolle sind also nicht erfüllt.

 

Lösung zur Aufgabe 7.3.2 - Anzahl der Nullstellen der Ableitung I

Die polynomielle Funktion \( f(x) \) dritten Grades ist stetig auf \( [0,2] \) und differenzierbar in \( (0,2). \) Ferner erfüllt sie \[ f(0)=0,\quad f(1)=0,\quad f(2)=0. \] Nach dem Satz von Rolle existieren also \( x_0\in(0,1) \) und \( x_1\in(0,2) \) mit \[ f'(x_0)=0,\quad f'(x_1)=0. \] Weitere Nullstellen der Ableitung existieren nicht, denn die Ableitung ist ein Polynom zweiten Grades, dessen Nullstellenmengen nach Aufgabe 7.2.1 höchstens zwei verschiedene Elemente besitzt. Das war zu zeigen.\( \qquad\Box \)

 

Lösung zur Aufgabe 7.3.3 - Anzahl der Nullstellen der Ableitung II

Es bezeichnen \( x_0\ge a \) die kleinste und \( x_1\le b \) die größte der \( n \) Nullstellen von \( f(x) \) in \( [a,b]. \) Das Intervall \( [x_0,x_1] \) zerfällt dann in \( n-1 \) Teilintervalle. In jedem dieser Teilintervalle liegt nach dem Satz von Rolle wenigstens eine Nullstelle der Ableitung \( f'(x). \) Bezeichnen wir also mit \( n' \) die Anzahl der Nullstellen von \( f'(x), \) so gilt \[ n'\ge n-1. \] Diese Ungleichung gilt jetzt aber auch in dem Ausgangsintervall \( [a,b], \) denn die Anzahl \( n' \) der Nullstellen von \( f'(x) \) kann durch Übergang von \( [x_0,x_1]\subseteq[a,b] \) zu \( [a,b] \) nicht kleiner werden, sondern bleibt gleich oder wächst sogar an. Umstellen liefert also \[ n\le n'+1, \] was zu zeigen war.\( \qquad\Box \)

 

Lösungen zu den Aufgaben Ein notwendiges Kriterium für lokale Extrema

 

Lösung zur Aufgabe 7.3.4 - Bestimmen absoluter Extrema

(i) Es ist \( f(x) \) streng monoton wachsend, d.h.

\[ \begin{array}{l} m=0\quad\mbox{mit}\ f(1)=m, \\ M=1\quad\mbox{mit}\ f(0)=1. \end{array} \]

(i) Im Fall dieser Parabelfunktion ermitteln wir

\[ \begin{array}{l} m=0\quad\mbox{mit}\ f(0)=0, \\ M=1\quad\mbox{mit}\ f(-1)=f(1)=1. \end{array} \]

(i) Wir ermitteln

\[ \begin{array}{l} m=-1\quad\mbox{mit}\ f(x)=-1\ \mbox{für}\ -1\le x\lt 0, \\ M=1\quad\mbox{mit}\ f(x)=1\ \mbox{für}\ 0\lt x\le 1. \end{array} \] Damit sind alle Größen ermittelt.\( \qquad\Box \)

 

Lösung zur Aufgabe 7.3.5 - Bestimmen von Extrema

(i) Wir berechnen

\[ f'(x)=2x-2, \]

  und damit

\[ f'(x)=0\quad\mbox{genau dann, wenn}\ x=1. \]

  An der Stelle \( x=1 \) befindet sich ein lokales Minimum, denn für \( \varepsilon\in\mathbb R\setminus\{0\} \) hinreichend klein ermitteln wir

\[ f(1+\varepsilon) =(1+\varepsilon)^2-2(1+\varepsilon)-8 =1+2\varepsilon+\varepsilon^2-2-2\varepsilon-8 =-9+\varepsilon^2 \gt -9. \]

  Ferner ist \( f(x) \) streng monoton fallend für \( -2\le x\lt 1 \) und streng monoton wachsend für \( 1\lt x\lt 2. \) Wegen \( f(-2)=0 \) und \( f(2)=-8\gt -9 \) erhalten wir zusammenfassend

\[ \begin{array}{l} \mbox{lokales und globales Minimum in}\quad x_0=1\ \mbox{mit}\ f(x_0)=-9 \\ \mbox{lokale Maxima in}\ x_1=-2\ \mbox{mit}\ f(x_1)=0\ \mbox{und}\ x_2=2\ \mbox{mit}\ f(x_2)=-8, \\ \mbox{gobales Maximum in}\quad x_1=-2\ \mbox{mit}\ f(x_1)=0. \end{array} \]

(ii) Wir ermitteln

\[ f'(x)=\frac{1-x^2}{(1+x^2)^2} \]

  und damit

\[ f'(x)=0\quad\mbox{genau dann, wenn}\ x\in\{-1,1\}\,. \]

  In \( x_0=-1 \) befindet sich ein lokales Minimum. Ist nämlich \( \varepsilon\gt 0 \) hinreichend klein, so dass \( \varepsilon\lt 1 \) und 2-2\vrepsilon+\varepsilon\gt 0, \) so ermitteln wir

\[ \begin{array}{lll} \varepsilon^2\gt 0 & \Longleftrightarrow & 2-2\varepsilon+\varepsilon^2\gt 2-\varepsilon \\ & \Longleftrightarrow & \displaystyle \frac{1}{2}\gt\frac{1-\varepsilon}{2-2\varepsilon+\varepsilon^2} \\ & \Longleftrightarrow & \displaystyle \frac{-1+\varepsilon}{2-2\varepsilon+\varepsilon^2}\gt-\,\frac{1}{2} \\ & \Longleftrightarrow & \displaystyle \frac{-1+\varepsilon}{1+(-1+\varepsilon)^2}\gt-\,\frac{1}{2} \\ & \Longleftrightarrow & \displaystyle f(-1+\varepsilon)\gt-\,\frac{1}{2} \end{array} \]

  Entsprechend behandelt man den Fall hinreichend kleines \( \varepsilon\gt 0. \) In \( x_1=1 \) befindet sich weiter ein lokales Maximum (gleiche Begründung wie eben). Die Funktion ist monoton fallend für \( -3\le x\lt -1, \) monoton wachsend für \( -1\le x\le 1 \) und monoton fallend für \( 1\lt x\le 3. \) Zusammen mit \( f(-3)=-\frac{3}{10} \) und \( f(3)=\frac{3}{10} \) erhalten wir zusammenfassend

\[ \begin{array}{l} \displaystyle\mbox{lokales und globales Minimum in}\quad x_0=-1\ \mbox{mit}\ f(x_0)=-\,\frac{1}{2}\,, \\ \displaystyle\mbox{lokale Minimum in}\ x_1=3\ \mbox{mit}\ f(x_1)=\frac{3}{10}\,, \\ \displaystyle\mbox{lokales und gobales Maximum in}\quad x_2=1\ \mbox{mit}\ f(x_2)=\frac{1}{2}\,, \\ \displaystyle\mbox{lokales Maximum in} x_4=-3\ \mbox{mit}\ f(x_4)=-\,\frac{3}{10}\,. \end{array} \]

 

Lösungen zu den Aufgaben Mittelwertsätze

 

Lösung zur Aufgabe 7.3.6 - Nepersche Ungleichung

Wir wenden den klassischen Mittelwertsatz auf die Funktion \( f(x)=\ln x \) an und erhalten \[ \frac{\ln b-\ln a}{b-a}=\frac{1}{x_0}\quad\mbox{mit einem}\ x_0\in(a,b). \] Dabei beachten wir \[ \frac{1}{b}\lt\frac{1}{x_0}\lt\frac{1}{a}\,, \] so dass folgt \[ \frac{1}{b}\lt\frac{\ln b-\ln a}{b-a}\lt\frac{1}{a}\,. \] Das ist die gesuchte Ungleichung.\( \qquad\Box \)

 

Lösung zur Aufgabe 7.3.7 - Folgerungen aus der Neperschen Ungleichung

(i) In die Nepersche Ungleichung setzen wir \( a=1 \) und \( b=x \) ein und erhalten

\[ \frac{1}{x}\lt\frac{\ln x-\ln 1}{x-1}\lt 1 \quad\mbox{bzw.}\quad \frac{x-1}{x}\lt\ln x\lt x-1 \]

  für alle \( x\in(1,\infty) \) unter Beachtung von \( \ln 1=0. \)
(ii) In die Nepersche Ungleichung setzen wir \( a=1 \) und \( b=1+x \) ein und erhalten

\[ \frac{1}{1+x}\lt\frac{\ln(1+x)-\ln 1}{1+x-1}\lt 1 \quad\mbox{bzw.}\quad \frac{x}{1+x}\lt\ln(1+x)\lt x \]

  für alle \( x\in(0,\infty). \)
(ii) In die Nepersche Ungleichung setzen wir \( a=1 \) und \( \displaystyle b=1+\frac{1}{x} \) ein und erhalten

\[ \frac{x}{1+x}\lt\frac{\ln\left(1+\frac{1}{x}\right)-\ln 1}{1+\frac{1}{x}-1}\lt 1 \quad\mbox{bzw.}\quad \frac{1}{1+x}\lt\ln\left(1+\frac{1}{x}\right)\lt\frac{1}{x} \]

  für alle \( x\in(0,\infty). \)

Damit ist alles gezeigt.\( \qquad\Box \)

 

Lösung zur Aufgabe 7.3.8 - Fehlerabschätzungen für den natürlichen Logarithmen

(i) In Aufgabe 7.3.8(i) setzen wir \( x=10 \) ein und erhalten

\[ \frac{1}{1+10}\lt\ln\left(1+\frac{1}{10}\right)\lt\frac{1}{10} \quad\mbox{bzw.}\quad \frac{1}{11}\lt\ln\frac{11}{10}=\ln 1.1\lt\frac{1}{10}\,. \]

(ii) In Aufgabe 7.3.8(i) setzen wir \( x=100 \) ein und erhalten

\[ \frac{1}{1+100}\lt\ln\left(1+\frac{1}{100}\right)\lt\frac{1}{100} \frac{1}{101}\lt\ln\frac{101}{100}=\ln 1.01\lt\frac{1}{100}\,. \]

(iii) In Aufgabe 7.3.8(i) setzen wir \( x=1000 \) ein und erhalten

\[ \frac{1}{1+1000}\lt\ln\left(1+\frac{1}{1000}\right)\lt\frac{1}{1000} \quad\mbox{bzw.}\quad \frac{1}{1001}\lt\ln\frac{1001}{1000}=\ln 1.001\lt\frac{1}{1000}\,. \] Damit sind alle Behauptungen gezeigt.\( \qquad\Box \)

 

Lösung zur Aufgabe 7.3.9 - Folgerung aus dem klassischen Mittelwertsatz I

Wir wenden den klassischen Mittelwertsatz an auf das Intervall \( (x_0,x_0+h)\subset(a,b), \) wobei \( x_0 \) eine andere Bedeutung besitzt als in der Vorlesung, und einem geeigneten Mittelwert \( x_0+\vartheta h \) mit \( \vartheta\in(0,1) \) entsprechend geeignet. Dann folgt \[ f'(x_0+\vartheta h)=\frac{f(x_0+h)-f(x_0)}{x_0+h-x_0}=\frac{f(x_0+h)-f(x_0)}{h}\,, \] und die Behauptung folgt nach Umstellen.\( \qquad\Box \)

 

Lösung zur Aufgabe 7.3.10 - Folgerung aus dem klassischen Mittelwertsatz II

(i) In Aufgabe 7.3.9 setzen wir \( x_0=0 \) und \( h=x\gt 0 \) und erhalten

\[ e^x=e^0+e^{\vartheta x}x. \]

  Das zeigt die Behauptung (i).
(ii) Die Exponentialfunktion ist streng monoton wachsend. Daher gilt einmal

\[ 1+xe^{0\cdot x}=1+x\le e^x \]

  für die Wahl von \( \vartheta\to 0, \) und zweitens für \( \vartheta\to 1 \) ist

\[ e^x\le 1+xe^{1\cdot x}=1+xe^x \quad\mbox{bzw.}\quad e^x\le\frac{1}{1-x}\,,\quad x\in(0,1). \] Damit ist alles bewiesen.\( \qquad\Box \)

 

Lösung zur Aufgabe 7.3.11 - Fehlerabschätzung der Wurzeln der Eulerschen Zahl

(i) In der Abschätzung aus Aufgabe 7.3.10(ii) setzen wir \( x=\frac{1}{2} \) und erhalten

\[ 1+\frac{1}{2}\lt e^\frac{1}{2}\lt\frac{1}{1-\frac{1}{2}} \quad\mbox{bzw.}\quad \frac{3}{2}\lt\sqrt{e}\lt 2. \]

(ii) In der Abschätzung aus Aufgabe 7.3.10(ii) setzen wir \( x=\frac{1}{3} \) und erhalten

\[ 1+\frac{1}{3}\lt e^\frac{1}{3}\lt\frac{1}{1-\frac{1}{3}} \quad\mbox{bzw.}\quad \frac{4}{3}\lt\sqrt[3]{e}\lt\frac{3}{2}\,. \]

(iii) In der Abschätzung aus Aufgabe 7.3.10(ii) setzen wir \( x=\frac{1}{4} \) und erhalten

\[ 1+\frac{1}{4}\lt e^\frac{1}{4}\lt\frac{1}{1-\frac{1}{4}} \quad\mbox{bzw.}\quad \frac{5}{4}\lt\sqrt[4]{e}\lt\frac{4}{3}\,. \] Damit sind alle Behauptungen gezeigt.\( \qquad\Box \)

 

Lösung zur Aufgabe 7.3.12 - Monotonieverhalten von Funktionen

(i) Es sei \( a\lt x_1\lt x_2\lt b. \) Der klassische Mittelwertsatz liefert wegen \( f'(x)\ge 0 \) in \( (a,b) \)

\[ f(x_2)-f(x_1)=f'(x_0)(x_2-x_1)\ge 0 \]

  mit \( x_0\in(x_1,x_2) \) geeignet. Also gilt

\[ f(x_2)\ge f(x_1). \]

  Das gilt nun für alle \( a\lt x_1\lt x_2\lt b, \) und nach Hinzunahme der Randpunkte \( x=a \) und \( x=b, \) was wegen der Stetigkeit nichts am Wachstumsverhalten ändert, folgt die Behauptung (i).
(ii) Es gelte nun

\[ f(x_2)\ge f(x_1)\quad\mbox{für alle}\ x_1,x_2\in[a,b]\ \mbox{mit}\ x_1\le x_2\,. \]

  Wähle nun zu zwei solchen \( x_1,x_2\in[a,b] \) mit \( x_1\lt x_2 \) Punkte \( x,x^*\in(x_1,x_2) \) mit \( x\not=x^*. \) Dann ist

\[ \frac{f(x^*)-f(x)}{x^*-x}\ge 0, \]

  und zwar unabhängig, \( x\lt x^* \) oder \( x^*\lt x \) auf Grund der vorausgesetzten Monotonie. Im Grenzfall \( x\to x^* \) folgt daher

\[ f'(x^*)=\lim_{x\to x^*,\ x\not=x^*}\frac{f(x^*)-f(x)}{x^*-x}\ge 0. \]

  Da \( x_1,x_2\in[a,b] \) und \( x^*\in(x_1,x_2) \) beliebig gewählt wurden, folgt die Behauptung (ii).

Damit ist alles bewiesen.\( \qquad\Box \)

 

Lösung zur Aufgabe 7.3.13 - Monotonie, Umkehrfunktion und Ableitungen

(ii) Für beliebige \( x,y\in[-1,1] \) mit \( x\lt y \) ermitteln wir

\[ \begin{array}{lll} f(y)-f(x)\negthickspace & = & \negthickspace\displaystyle \frac{y}{1+y^2}-\frac{x}{1+x^2} \,=\,\frac{y(1+x^2)-x(1+y^2)}{(1+x^2)(1+y^2)} \\ & = & \negthickspace\displaystyle \frac{y-x}{(1+x^2)(1+y^2)}+\frac{xy(x-y)}{(1+x^2)(1+y^2)} \\ & = & \negthickspace\displaystyle (1-xy)\,\frac{y-x}{(1+x^2)(1+y^2)} \,\gt\,0 \end{array} \]

  denn es ist \( |xy|\le 1, \) wobei \( |xy|=1 \) nur im Fall \( x=-1, \) \( y=1 \) mit \( f(-1)\lt f(1). \) Also existiert existiert die Inverse \( g(y). \)
(iii) Es ist zunächst \( f(0)=0 \) bzw. \( g(0)=0. \) Weiter berechnen wir

\[ f'(x)=\frac{1}{1+x^2}-\frac{2x^2}{(1+x^2)^2}=\frac{1-x^2}{(1+x^2)^2} \]

  und damit \( f'(0)=1. \) Also erhalten wir \( g'(0)=1. \)
(iv) Es muss

\[ y=\frac{x}{1+x^2}\,\quad x\in[-1,1], \]

  nach \( x=g(y) \) umgestellt werden:

\[ g(y) =\left\{ \begin{array}{cl} \displaystyle \frac{1}{2y}+\sqrt{\frac{1}{4y^2}-1}\,, & \displaystyle -\,\frac{1}{2}\le y\lt 0 \\ 0, & y=0 \\ \displaystyle \frac{1}{2y}-\sqrt{\frac{1}{4y^2}-1}\,, & \displaystyle 0\lt y\lt\frac{1}{2} \end{array} \right. \] Damit ist alles gezeigt.\( \qquad\Box \)

 

Lösung zur Aufgabe 7.3.14 - Die Inversen der hyperbolischen Funktionen

(i) Wir berechnen

\[ \frac{d}{dx}\,\sinh x=\cosh x,\quad \frac{d}{dy}\,\cosh x=\sinh x. \]

  Wegen

\[ \cosh x\ge 1\quad\mbox{in}\ \mathbb R\quad\mbox{mit}\ \cosh 0=1,\ \mbox{sonst}\ \cosh x\gt 1, \]

  ist \( \sinh x \) streng monoton wachsend auf \( \mathbb R. \) Weiter ist

\[ \sinh x\lt 0\quad\mbox{in}\ (-\infty,0),\quad \sinh 0=0,\quad \sinh x\gt 0\quad\mbox{in}\ (0,\infty), \]

  d.h. \( \cosh x \) fällt streng monoton für \( x\lt 0 \) und wächst streng monoton für \( x\gt 0. \)
(ii) Für \( \mbox{arsinh} \) haben wir

\[ \begin{array}{l} \mbox{arsinh}\colon\mathbb R\longrightarrow\mathbb R, \\ \mbox{Definitionsbereich:}\ (-\infty,+\infty),\quad \mbox{Wertebereich:}\ (-\infty,+\infty), \end{array} \]

  und für \( \mbox{arcosh} \) haben wir (wir betrachten hier nur einen Ast)

\[ \begin{array}{l} \mbox{arcosh}\colon[1,\infty)\longrightarrow[0,\infty), \\ \mbox{Definitionsbereich:}\ [1,+\infty),\quad \mbox{Wertebereich:}\ [0,\infty) \end{array} \]

(iii) Für \( y=\sinh x \) ermitteln wir mit Aufgabe 7.1.14(ii)

\[ \frac{d}{dy}\,\mbox{arsinh}\,y =\frac{1}{\frac{d}{dx}\,\sinh x} =\frac{1}{\cosh x}=\frac{1}{\sqrt{1+\sinh^2x}}=\frac{1}{\sqrt{1+y^2}}\,, \]

  für \( y=\cosh x \) ermitteln wir

\[ \frac{d}{dy}\,\mbox{arcosh}\,y =\frac{1}{\frac{d}{dx}\,\cosh x} =\frac{1}{\sinh x} =\frac{1}{\sqrt{\cosh^2x-1}} =\frac{1}{\sqrt{y^2-1}}\,. \] Damit ist alles gezeigt.\( \qquad\Box \)

 

Lösung zur Aufgabe 7.3.15 - Hyperbolische Funktionen und natürlicher Logarithmus

(i) Wir stellen

\[ y=\sinh x=\frac{1}{2}\,(e^x-e^{-x}) \]

  nach \( x \) wie folgt um

\[ 2y=e^x-e^{-x} \quad\mbox{bzw.}\quad 2ye^x=e^{2x}-1 \quad\mbox{bzw.}\quad e^{2x}-2ye^x-1=0 \]

  mit der Lösung (achte auf das richtige Vorzeichen)

\[ e^x=y+\sqrt{y^2+1} \quad\mbox{bzw.}\quad x=\ln\left(y+\sqrt{y^2+1}\right),\quad y\in\mathbb R. \]

  Nach Umbenennen der Variablen \( x \) und \( y \) bedeutet das

\[ \mbox{arsinh}\,x=\ln\left(x+\sqrt{x^2+1}\right),\quad x\in\mathbb R. \]

(ii) Wir stellen

\[ y=\cosh x=\frac{1}{2}\,(e^x+e^{-x}) \]

  nach \( x \) wie folgt um

\[ 2y=e^x+e^{-x} \quad\mbox{bzw.}\quad 2ye^x=e^{2x}+1 \quad\mbox{bzw.}\quad e^{2x}-2ye^x+1=0 \]

  mit der Lösung (wir betrachten nur den einen Ast)

\[ e^x=y+\sqrt{y^2-1} \quad\mbox{bzw.}\quad x=\ln\left(y+\sqrt{y^2-1}\right),\quad y\in[1,\infty). \]

  Nach Umbenennen der Variablen \( x \) und \( y \) bedeutet das

\[ \mbox{arcosh}\,x=\ln\left(x+\sqrt{x^2-1}\right),\quad x\in[1,\infty). \] Damit ist alles gezeigt.\( \qquad\Box \)

 

Lösung zur Aufgabe 7.3.16 - Eine weitere Anwendung des Mittelwertsatzes

Angenommen, es existieren \( a\lt x_1\lt x_2\lt b \) mit \( f(x_1)\lt f(x_2). \) Der klassische Mittelwertsatz liefert dann für \( x_0\in(x_1,x_2) \) geeignet \[ f'(x_0)=\frac{f(x_2)-f(x_1)}{x_2-x_1}\gt 0 \] im Widerspruch zur Voraussetzung \( f'(x)=0 \) für alle \( x\in(a,b). \) Entsprechend folgt im anderen Fall \( f(x_1)\gt f(x_2) \) mit \( x_0\in(x_1,x_2) \) geeignet \[ f'(x_0)=\frac{f(x_2)-f(x_1)}{x_2-x_1}\lt 0 \] im Widerspruch zur Voraussetzung. Also gilt notwendig \( f(x)=\mbox{const.}\qquad\Box \)

 

Lösung zur Aufgabe 7.3.17 - Eindeutigkeitssatz der Differentialrechnung

Wir setzen \[ h(x):=g(x)-f(x),\quad x\in[a,b], \] und gehen in zwei Schritten vor:

\( \circ \) Es sei \(f'(x)=g'(x) \) bzw. \( h'(x)=0 \) in \( (a,b). \) Aufgabe 7.3.16 und Stetigkeit liefern

\[ h(x)=\mbox{const}\quad\mbox{bzw.}\quad f(x)=g(x)+\mbox{const}\quad\mbox{in}\ [a,b]. \]

\( \circ \) Sei \( f(x)=g(x)+C \) in \( (a,b) \) mit \( C\in\mathbb R. \) Ableiten ergibt bereits die Behauptung

\[ f'(x)=g'(x)+0=g'(x)\quad\mbox{in}\ (a,b). \] Damit ist alles gezeigt.\( \qquad\Box \)

 

Lösung zur Aufgabe 7.3.18 - Gegenbeispiel zum Mittelwertsatz I

Betrachte beispielsweise \( x=-\frac{1}{2}, \) \( y=\frac{1}{2}. \) Dann ist \[ \frac{f(y)-f(x)}{y-x}=\frac{0}{1}=0, \] es existiert aber kein \( x\in\left(-\frac{1}{2},\frac{1}{2}\right) \) mit \( f'(x)=0. \) Der klassische Mittelwertsatz kann hier also nicht angewendet werden.\( \qquad\Box \)

 

Lösung zur Aufgabe 7.3.19 - Gegenbeispiel zum Mittelwertsatz II

Wir ermitteln beispielsweise \[ \frac{f(1)-f(-1)}{1-(-1)}=\frac{1-(-1)}{1-(-1)}=1. \] Es existiert aber kein \( x\in(-1,1) \) mit \( f'(x)=1. \) Der klassische Mittelwertsatz kann hier also nicht angewendet werden.\( \qquad\Box \)

 

Lösungen zu den Aufgaben Ein hinreichendes Kriterium für strenge lokale Minima und Maxima

 

Lösung zur Aufgabe 7.3.20 - Strenges lokales Minimum der Parabel

Für \( f(x)=x^2 \) berechnen wir \[ f'(x)=2x,\quad f''(x)=2. \] Im Punkt \( x_0=0 \) sind damit die Voraussetzungen des Satzes aus Paragraph 7.3.4 erfüllt: \( f'(0)=0 \) und \( f'(0)\gt 0, \) so dass \( f(x) \) in \( x_0=0 \) ein strenges lokales Minimum besitzt.

 

Lösung zur Aufgabe 7.3.21 - Eine Extremwertaufgabe

Es ist \( L\gt 0 \) vorgegeben. Bezeichnet \( U_1 \) den Umfang des Kreises, so berechnet sich sein Flächeninhalt zu \[ A_1=\frac{U_1^2}{2\pi}\,. \] Bezeichnet weiter \( U_2 \) den Umfang des Quadrats, so ergibt sich sein Flächeninhalt zu \[ A_2=\left(\frac{U_2}{4}\right)^2\,. \] Es ist also die Funktion \( f\colon[0,1]\to[0,\infty) \) vermöge \[ f(x)=\frac{(xL)^2}{16}+\frac{[(1-x)L]^2}{2\pi}=L^2\left(\frac{1}{16}+\frac{1}{2\pi}\right)x^2-\frac{L}{\pi}\,x+\frac{L^2}{2\pi} \] zu maximieren. Nun ist \[ f''(x)=2L^2\left(\frac{1}{16}+\frac{1}{2\pi}\right)\gt 0, \] d.h. \( f(x) \) besitzt höchstens strenge lokale Minima. Da aber die stetige Funktion \( f(x) \) auf dem Kompaktum \( [0,1] \) ihr globales Minimum und ihr globales Maximum annimmt, kann letzteres nur in \( x_0=0 \) oder \( x_1=1 \) geschehen. Wegen \[ f(0)=\frac{L^2}{2\pi}\gt\frac{L^2}{16}=f(1) \] wird also der Gesamtinhalt maximal in \( x_0=0, \) d.h. wenn man den gesamten Draht für den Kreis verwendet.\( \qquad\Box \)